Putnam Marathon
by djmathman, Dec 31, 2020, 5:20 PM
Differential geometry destroyed me last semester, but at least I still had time to do some vintage Putnam.
Joy of Mathematics. Let
. Prove that
![\[
2\sin\alpha + \tan\alpha \geq 3\alpha.
\]](//latex.artofproblemsolving.com/3/1/2/312a08e810c59b737f37cf4922ae0472385da15e.png)
Cute
Putnam 1965 B3. Find all twice differentiable functions
satisfying
![\[
f(x)^2 - f(y)^2 = f(x-y)f(x+y).
\]](//latex.artofproblemsolving.com/0/6/7/0679da38da7c678fac5782ad6f983c616fa36645.png)
Sketch
Putnam 1968 A6. Determine all polynomials of the form
with
(
,
) such that each has only real zeros.
Solution
Putnam 1968 B6. A set of real numbers is compact if it is closed and bounded. Show that there does not exist a sequence
of compact sets of rational numbers such that each compact set of rationals is contained in at least one
.
Solution (slightly optimized from my own)
Putnam 1969 A6. Let a sequence
be given, and let
,
. Suppose that the sequence
converges. Prove that the sequence
also converges.
Surprisingly anticlimatic?
Putnam 1972 A6. Let
be an integrable function in
and suppose
,
,
,
and
. Show that
in a set of positive measure.
lol
Putnam 1972 B4. Let
be an integer greater than
. Show that there exists a polynomial
with integral coefficients such that
.
Sketch
Putnam 1972 B6. Let
be a set of positive integers. Prove that the polynomial
has no roots inside the circle
.
Solution
Putnam 1976 A5. In the
plane, if
is the set of points inside and on a convex polygon, let
be the distance from
to the nearest point of
. (a) Show that there exist constants
,
, and
, independent of
, such that
where
is the perimeter of
and
is the area of
. (b) Find the values of
,
, and
.
I love this
Putnam 1976 B6. As usual, let
denote the sum of all the (positive integral) divisors of
. For example, if
is a prime, then
. Motivated by notion of a "perfect" number, a positive integer
is called "quasiperfect" if
. Prove that every quasiperfect number is the square of an odd integer.
Solution
Steinhaus. Let
be a Lebesgue measurable set in
with
. Then the difference set
contains a nonzero neighborhood of the origin.
Oops I did this a while ago
Joy of Mathematics. Let

![\[
2\sin\alpha + \tan\alpha \geq 3\alpha.
\]](http://latex.artofproblemsolving.com/3/1/2/312a08e810c59b737f37cf4922ae0472385da15e.png)
Cute
Let
. Observe that
and
where the last inequality is by AM-GM. Thus
by the Fundamental Theorem of Calculus.


![\[
f'(\alpha) = 2\cos\alpha + \sec^2\alpha = \cos \alpha + \cos\alpha + \frac{1}{\cos^2\alpha} \geq 3,
\]](http://latex.artofproblemsolving.com/b/b/3/bb341dc6e5cd4aa9071eade2dac8b87004e00b87.png)

Putnam 1965 B3. Find all twice differentiable functions

![\[
f(x)^2 - f(y)^2 = f(x-y)f(x+y).
\]](http://latex.artofproblemsolving.com/0/6/7/0679da38da7c678fac5782ad6f983c616fa36645.png)
Sketch
Differentiate the given with respect to
to get
Now observe that the RHS is independent of
since the LHS is by default. That is, we may reparametrize to get the existence of a differentiable function
satisfying
Differentiate wrt
and get cross terms to cancel; this yields
. Hence either
or
for some constant
.
If
you get linear, if
you get exponential (hyperbolic trig), and if
you get trigonometric.

![\[
2f(x)f'(x) = f(x+y)f'(x-y) + f'(x+y)f(x-y).
\]](http://latex.artofproblemsolving.com/5/3/b/53b9a4a47eef0abc46c6269da28e5af63ba2b2a3.png)


![\[
f(x)f'(s-x) + f'(x)f(s-x) =: g(s).
\]](http://latex.artofproblemsolving.com/d/5/b/d5bef9360dd3155df714e14e897aa6f398eeec3c.png)





If



Putnam 1968 A6. Determine all polynomials of the form




Solution
Assume
without loss. We claim that the only polynomials of this form are
,
,
,
,
, and
(whoops missed these last two possibilities at first). These polynomials trivially work, so it remains to show they are the only ones.
Let
,
,
denote the roots of the given polynomial. By Vieta,
This expression is negative when
, so
, which implies
. But by AM-GM,
ergo,
.
We now handle each case in turn. The case
is easy since both possible polynomials are linear. The case
is easy by manually checking the two possibilities. Finally, in the case
, we have equality in
, so
for all
; another quick check of the four possible cases yields the two shown above.







Let



![\[
\sum_{i=1}^nr_i^2 = a_1^2 - 2a_2 = 1 - 2a_2.
\]](http://latex.artofproblemsolving.com/d/b/d/dbd3d4d3e6d7b2927429edcb4ebf3d3610dac67a.png)



![\[
\frac 3n = \frac 1n\sum_{i=1}^nr_i^2 \geq \left(\prod_{i=1}^nr_i^2\right)^{1/n} = 1;\quad(*)
\]](http://latex.artofproblemsolving.com/a/b/e/abec31da813f73a60654bb2a778132d6f6ed2dcb.png)

We now handle each case in turn. The case






Putnam 1968 B6. A set of real numbers is compact if it is closed and bounded. Show that there does not exist a sequence


Solution (slightly optimized from my own)
A standard diagonalization-type argument does the job.
Fix such a sequence
of compact sets. For each positive integer
, set
. Let
be any irrational number in the interval
. Observe that there exists
such that
, or else
would be a limit point of the set
(contradicting the fact that
is closed). As a result, we may choose
.
Now set
. Then
is a bounded set of rational numbers whose sole limit point is
-- hence
is compact -- and
for each
.
Fix such a sequence











Now set






Putnam 1969 A6. Let a sequence





Surprisingly anticlimatic?
Let
; by subtracting
from each
we may assume
. We now claim
as
; this finishes the proof.
Let
be arbitrary. Observe that, since
as
, there exists
such that
for all
. For these values of
,
Thus
, and so
for sufficiently large
.






Let







![\[
|x_n| = \frac{|y_n - x_{n-1}|}2 \leq \frac{|y_n| + |x_{n-1}|}2 < \frac{\tfrac \varepsilon 2 + |x_{n-1}|}2.
\]](http://latex.artofproblemsolving.com/2/3/9/239da847f7cd8faebf52d40b853d3d9bca3d59f3.png)

![\[
|x_n| < \frac\varepsilon 2 + \frac{|x_N|}{2^{n-N}} < \varepsilon
\]](http://latex.artofproblemsolving.com/b/4/3/b43806d0337812b0008945d55e19c064f7e9fd9d.png)

Putnam 1972 A6. Let








lol
If not, then
this took forever >.<

Putnam 1972 B4. Let




Sketch
We will instead generalize the problem massively and show there exists a polynomial
with integer coefficients such that
where
,
, and
are positive integers with
. Two steps.






- Start by writing
Inductively push the minimum degree of the nonlinear terms higher and higher.
- Sylvester tells you that every monomial
, where
, is of the form
for nonnegative integers
and
. Use this to clean up the high degree terms.
Putnam 1972 B6. Let



Solution
Let
.If
then we are trivially done, so assume
. There are two cases to consider.
In the first case, suppose
, and let
be a complex number satisfying
. Then
This means
, or
.
In the second case, suppose
, and again assume
is a root of the given polynomial. Then
where each
is either
,
, or
. As a result, analogous reasoning as in the previous case yields
leading to the same bound as before.
We have exhausted both cases, and so we are done.



In the first case, suppose






In the second case, suppose


![\[
(1-z)P(z) = (1-z)\left(1 + z + \sum_{j=2}^kz^{n_j}\right) = 1 - z^2 + \sum_{m=3}^{n_k + 1}\varepsilon_mz^m,
\]](http://latex.artofproblemsolving.com/c/b/6/cb64d993024880cea9bf8304202271cdcaab9c53.png)




![\[
1 = \left|z^2 - \sum_{m=3}^{n_k+1}\varepsilon_mz^m\right| \leq |z|^2 + |z|^3 + \cdots = \frac{|z|^2}{1 - |z|},
\]](http://latex.artofproblemsolving.com/d/5/8/d58620da71f6d7ed996e5e8a72a44a7bbd9154b9.png)
We have exhausted both cases, and so we are done.
Putnam 1976 A5. In the









![\[
\int_{-\infty}^\infty \int_{-\infty}^\infty e^{-D(x,y)}\,dx\,dy = a + bL + cA,
\]](http://latex.artofproblemsolving.com/f/d/8/fd8c84800620fec871b09a515fed29f3d9627aac.png)







I love this
Let
be any given convex polygon with sides
and vertices
,
,
. For each side
of
, we may erect rays
and
through the endpoints of
perpendicular to
and exterior to
. Let
denote the region bounded by
,
, and
for each
. These regions are disjoint and divide the remains of
into several unbounded regions
, one per vertex.
We now split our desired integral over
into several parts by writing
There are three cases to consider.
,
, and
in the expression above.



















We now split our desired integral over


- Within the region of integration
,
is always zero, so this integral equals
.
- Within the region of integration
,
is simply the distance from
to the segment
. The given integral is invariant under translation and rotation of
, so assume without loss that
is the segment connecting
to
. Then
, and so
Summing over all
yields that
- Within the region of integration
,
is simply the distance from
to the vertex
. The given integral is invariant under translation of
, so we may translate each of the vertices
to coincide at the origin. But now
is actually the entire plane
, since the sum of the measures of all the vertex angles is
. Therefore



Putnam 1976 B6. As usual, let






Solution
This solution isn't entirely mine, in the sense that I misremembered the problem before checking my solution on the Kalva Putnam archive. While I did end up figuring out the rest of it, I was definitely influenced by the bits and pieces I saw of that Kalva solution.
Recall that, if
, then
Now each factor
is odd if and only if either
or (in the case where
)
is even. Thus, since
is odd, we must have
for some odd positive integer
.
Plugging this back into the original equation yields
Now suppose
. Then
, so there exists a prime
with
. Then
contradiction since
. Thus,
and
is the square of an odd integer.
Recall that, if

![\[
\sigma(N) = \sigma(p_1^{e_1})\cdots\sigma(p_k^{e_k}) = (1+p_1 + \cdots + p_1^{e_1})\cdots (1+p_k + \cdots + p_k^{e_k}).
\]](http://latex.artofproblemsolving.com/4/1/f/41f70b6b14cd8e6f321774eb1fa47abcb972c092.png)







Plugging this back into the original equation yields
![\[
(2^{a+1} - 1)\sigma(m^2) = 2^{a+1}m^2 + 1.
\]](http://latex.artofproblemsolving.com/f/e/5/fe53f78e761d384dcbb197d7fad4afb7327b9eca.png)




![\[
2^{a+1}m^2 + 1 \equiv m^2 + 1 \equiv 0\pmod p,
\]](http://latex.artofproblemsolving.com/f/3/3/f33c963a6f8469ca9862f6dfba7a5c5e1f2999db.png)



Steinhaus. Let



![\[
A - A \coloneqq \{x - y: x, y\in A\}
\]](http://latex.artofproblemsolving.com/1/b/7/1b74b5ace07134e09db61ea6a5a33b356e1ea122.png)
Oops I did this a while ago
We proceed in steps.
Step 1. First suppose
is both bounded and Borel measurable. We need a lemma.
Lemma. Let
. Suppose
is a bounded Borel measurable set such that
for every interval
. Then
.
Proof. For every collection of intervals
satisfying
, we have
Taking the infimum of the right hand side over all such collections of intervals yields
, and since
we deduce
. 
Returning to the original problem, note that by the lemma we may choose an interval
such that
. Without loss of generality, assume
, and let
and
. Then, for all
, we have
Hence there exists some
, so
. In turn,
.
Step 2. Now suppose
is simply Borel measurable (but not necessarily bounded). Then
so there exists
such that
. Now apply the previous step to
.
Step 3. Finally, assume
is Lebesgue measurable. It is known that there exists a closed set
such that
. But closed sets are Borel measurable, and so we may apply Step 2 to
.
Step 1. First suppose

Lemma. Let


![\[
m(A\cap I)\leq \lambda m(I)
\]](http://latex.artofproblemsolving.com/6/b/e/6be24b9bbd098a8851eff4670863c82985347e96.png)


Proof. For every collection of intervals


![\[
m(A) \leq \sum_{n\geq 1}m(A\cap I_n)\leq \sum_{n\geq 1}\lambda m(I_n) = \lambda\sum_{n\geq 1}m(I_n).
\]](http://latex.artofproblemsolving.com/0/f/4/0f47c6bb313747d570b4b77fdd5a526740bf7755.png)




Returning to the original problem, note that by the lemma we may choose an interval


![$I = [0,1]$](http://latex.artofproblemsolving.com/1/f/c/1fce074fdcba0b6f4deeb98871c80f60d51fb443.png)



![\begin{align*}
m(B\cap (B+\delta)) &= m(B) + m(B+\delta) - m(B\cup(B+\delta)) \\
&\geq 2(\varepsilon + \tfrac12) - m([1,1+\varepsilon]) = \varepsilon.
\end{align*}](http://latex.artofproblemsolving.com/9/3/f/93f54d82f86762dcc1fb1afa02db47e17fcb0d8f.png)



Step 2. Now suppose

![\[
m(A) = \lim_{n\to\infty}m(A\cap[-n,n]),
\]](http://latex.artofproblemsolving.com/a/8/d/a8db06a8c73e36d540bc7b573b26fbe8e0a95e26.png)

![$m(A\cap[-n,n]) > 0$](http://latex.artofproblemsolving.com/8/c/c/8ccec2c3fe62b4c3e148ab76a96d13a893a229cf.png)
![$A\cap[-n,n]$](http://latex.artofproblemsolving.com/a/6/8/a682a3e9e6519935a556edf7fec755d0aee193d8.png)
Step 3. Finally, assume




This post has been edited 1 time. Last edited by djmathman, Dec 31, 2020, 5:36 PM